The terminal side of an angle in standard position passes through p(–3, –4). what is the value of tangent theta?

Answers

Answer 1

Answer:

  4/3

Step-by-step explanation:

The tangent of any angle (θ) in standard position that has point (x, y) on its terminal ray is ...

  tan(θ) = y/x

__

For the given point on the terminal side, the tangent is ...

  tan(θ) = (-4)/(-3) = 4/3

_____

Additional comment

There are several ways this can be explained. One of them makes use of the relation between rectangular and polar coordinates:

  (x, y) = (r·cos(θ), r·sin(θ))

Then the ratio y/x is ...

  y/x = (r·sin(θ))/(r·cos(θ)) = sin(θ)/cos(θ) = tan(θ)


Related Questions

The correlation in error terms that arises when the error terms at successive points in time are related is termed _____.

Answers

Answer:

auto correlation is the answer to the question

A company making tires for bikes is concerned about the exact width of its cyclocross tires. The company has a lower specification limit of 22.8 millimeters and an upper specification limit of 23.1 millimeters. The standard deviation is 0.19 millimeters and the mean is 22.9 millimeters. What is the process capability index for the process? Note: Round your answer to 4 decimal places.

Answers

The process capability index for the process is 0.1754.

How to calculate the index?

The first sided specification limit will be:

= (Upper specification limit - mean)/(3 × standard deviation)

= (23.1 - 22.9)/(3 × 0.19)

= 0.2/0.57

= 0.3508

The second sided specification limit will be:

= (22.9 - 22.8)/(3 × 0.19)

= 0.1/0.57

= 0.1754

The process capability index for the process is 0.1754 wine it's the lower value.

Learn more about capability index on:

brainly.com/question/15734839

#SPJ12

Someone help me with this math word questions

Answers

2) The three ways of naming an angle are by the letter at the vertex, by placing a number or variable inside the angle, or by two letters with the vertex letter placed in the middle.

3) [tex]\angle WXY, \angle YXZ, \angle WXZ[/tex]

4) [tex]X[/tex]

Given: l || m; ∠1 ∠3

Prove: p || q

Horizontal and parallel lines l and m are intersected by parallel lines p and q. At the intersection of lines l and p, the uppercase left angle is angle 1. At the intersection of lines q and l, the bottom right angle is angle 2. At the intersection of lines q and m, the uppercase left angle is angle 3.

Complete the missing parts of the paragraph proof.



We know that angle 1 is congruent to angle 3 and that line l is parallel to line m because
. We see that is congruent to by the alternate interior angles theorem. Therefore, angle 1 is congruent to angle 2 by the transitive property. So, we can conclude that lines p and q are parallel by the

.

Answers

The parts that are missing in the proof are:

It is given

∠2 ≅ ∠3

converse alternate exterior angles theorem

What is the Converse of Alternate Exterior Angles Theorem?

The theorem states that, if two exterior alternate angles are congruent, then the lines cut by the transversal are parallel.

∠1 ≅ ∠3 and l║m because we are: given

By the transitive property,

∠2 and ∠3 are alternate interior angles, therefore, they are congruent to each other by the alternate interior angles theorem.

Based on the converse alternate exterior angles theorem, lines p and q are proven to be parallel.

Therefore, the missing parts pf the paragraph proof are:

It is given∠2 ≅ ∠3converse alternate exterior angles theorem

Learn more about the converse alternate exterior angles theorem on:

https://brainly.com/question/17883766

#SPJ1

Answer:

it is given

angle 2

angle 3

converse alternate exterior angles theorem

Step-by-step explanation:

Add.

(3+x3+3x2)+(2x3−2−4x2)



Express the answer in standard form.

Enter your answer in the box.

Answers

Step-by-step explanation:

(3+3+3*2)+(2*3-2-4*2)

The value of the addition of the two polynomials is 3x³ - x² + 1.

What is addition of polynomial?

The addition of polynomials involves combining like terms of two or more polynomials to create a new polynomial. A polynomial is an expression consisting of variables, coefficients, and exponents, connected by addition and subtraction operations.

To add the given polynomials:

(3 + x³ + 3x²) + (2x³ - 2 - 4x²)

First, let's group the like terms:

(x³ + 2x³) + (3x² - 4x²) + (3 - 2)

Combine the coefficients of the like terms:

3x³ + 3x² - 4x² + 3 - 2

Simplify the expression:

3x³ - x² + 1

Therefore, the sum of the given polynomials is: 3x³ - x² + 1.

Learn more on addition of polynomial here;

https://brainly.com/question/10078818

#SPJ2

f(x) = x + 6 , find the ordered pair when x = -2.

(-2,4)

(2,-4)

(-2,-4)

(2,4)

Answers

-------------------------------------------------------------------------------------------------------------

Answer:  [tex]\textsf{Option A, (-2, 4)}[/tex]

-------------------------------------------------------------------------------------------------------------

Given:  [tex]\textsf{f(x) = x + 6}[/tex]

Find:  [tex]\textsf{f(-2)}[/tex]

Solution:  We need to plug in the value -2 into every variable x that we have in the expression and simplify to get the solution.

Plug in the values

[tex]\textsf{f(x) = x + 6}[/tex][tex]\textsf{f(-2) = -2 + 6}[/tex]

Simplify

[tex]\textsf{f(-2) = -2 + 6}[/tex][tex]\textsf{f(-2) = 4}[/tex]

Therefore, the ordered pair that would match would have a x-coordinate of -2 and a y-coordinate of 4 producing (-2, 4) which matches option A.

What is a 10% margin increase on 1,810.69?

Answers

Answer:

1991.759

Step-by-step explanation:

1810.69×%10=181.069

1810.69+181.069=1991.759

Please Help me!!!!!!!!!

Answers

Answer:

(2,-3) (3,-3)

Step-by-step explanation:

f(x) is y

x² - 5x + 3 = -3

x² - 5x + 6 = 0

ask yourself

which 2 numbers when multiplied give you

+6 and when added give you -5

they are -3 and -2

(x-3)(x-2)

x=3 x=2

when you put x=3 and x=2

in x² - 5x + 3 it's gives -3 which is y

(3,-3) (2,-3)

https://brainly.com/question/21836835

1 in = 2.54 cm
how many millimeters are in 10.5 feet?
A.266.7 mm
B. 1,260 mm
C. 320.04 mm
D. 3,200.4 mm

Answers

Answer:

[tex]\fbox {D. 3,200.4 mm}[/tex]

Step-by-step explanation:

Given :

[ 1 inch = 2.54 centimeters ]

Unit conversions to keep in mind :

1 feet = 12 inches1 cm = 10 mm

Solving

10.5 feet10.5 x 12 inches126 inches126 x 2.54 cm320.04 cm320.04 x 10 mm3200.4 mm
The answer is D 3,200.4 mm I get that answer be equal ovulating the millimeters by 10.5 feet divided by the equal force of us in typical jetstream. Therefore 1 inches 2.54 cm. Then if I divide that by two, I get 3,200.4.

f(x)=(2x−3)(x+6)(5x+6)f, left parenthesis, x, right parenthesis, equals, left parenthesis, 2, x, minus, 3, right parenthesis, left parenthesis, x, plus, 6, right parenthesis, left parenthesis, 5, x, plus, 6, right parenthesis has zeros at x=-6x=−6x, equals, minus, 6, x=-\dfrac{6}{5}x=− 5 6 ​ x, equals, minus, start fraction, 6, divided by, 5, end fraction, and x=\dfrac{3}{2}x= 2 3 ​ x, equals, start fraction, 3, divided by, 2, end fraction. What is the sign of fff on the interval -6

Answers

Answer:

566

Step-by-step explanation:

I got it right..................

Please help me out…

Polygon ____ and polygon ____ are similar to polygon 1.

Answers

Polygon 3 and 4 are similar to polygon 1.

Polygons will be called similar if they are exactly in the same shape, but can be in different sizes.

Here given, Some of the images are images of the polygon 1 from similarity transformations.

So we have to find the similar polygon from the given picture.

From picture it is clear that Polygon 2 is similar to polygon 1 as the size of polygon 2 has been decreased from polygon 1 but the shape of polygon 2 is the same as polygon 1 as we can their corresponding angles are also equal.

Also, polygon 3 is similar to polygon 1 as the size of polygon 3 has been decreased from polygon 1 but the shape of polygon 2 is the same as polygon 1 and reversed of polygon 1. So polygon 3 is of the same orientation as polygon 1 as we can their corresponding angles are also equal.

So from above, it is clear that Polygon 1 is similar to polygon 3 and 4 as they have the same shape but different sizes.

Therefore Polygon 3 and Polygon 4 are similar to polygon 1.

Learn more about similar polygon

here: https://brainly.com/question/1493409

#SPJ10

[tex](1/8)^-^3^a=512^3^a[/tex]

Answers

The equation [tex](\frac18)^{-3a} = 512^{3a}[/tex] as no solution

How to solve the equation?

The equation is given as:

[tex](\frac18)^{-3a} = 512^{3a}[/tex]

Express the equation as a base of 2

[tex](2^{-3})^{-3a} = 2^{9*3a}[/tex]

Evaluate the products

[tex]2^{9a} = 2^{27a}[/tex]

Cancel out the common base

9a = 27a

The above equation is not true because [tex]9a \ne 27a[/tex]

Hence, the equation [tex](\frac18)^{-3a} = 512^{3a}[/tex] as no solution

Read more about equations at:

https://brainly.com/question/2456547

#SPJ1

During a catered lunch, an average of 4 cups of tea are poured per minute. The lunch will last 2 hours. How many gallons of tea should the caterer bring if there are 16 cups in one gallon?

Answers

Answer:

30

Step-by-step explanation:

4 cups x 60 hours a min x 2 hours of lunch ÷ 16 cups per gallon = 30 gallons

The answer is 30 gallons.

Firstly, convert the hours to minutes so you know how many minutes you're working with. 2*60=120 minutes.

Then, multiply 120 with 4 to get the number of tea (in cups) that is poured. 120*4=480 cups.

Finally, divide 480 cups with 16 cups to find out the number of gallons poured. 480/16 equals 30 gallons.

Hope this helped.

The quadrilateral below is formed from a parallelogram and an
isosceles triangle.
Calculate the size of angle VQU.

Answers

Answer:

VQU =58°

Step-by-step explanation:

angle QUV is 61 because of F angles (I don't know what they call them in ur country)

triangle QUV is an isosceles triangles so base angles are the same

61+61=122

180-122=58

What is the sum of the arithmetic sequence 3, 9, 15..., if there are 24 terms? (5 points)

Answers

There's a fixed difference of 6 between terms (9 - 3 = 6, 15 - 9 = 6, and so on). The first term of the sequence is 3, so the [tex]n[/tex]-th term is

[tex]3 + 6(n-1) = 6n - 3[/tex]

If there are 24 terms in the sum, then the last term is 6×23 - 3 = 135.

Let [tex]S[/tex] be the sum,

[tex]S = 3 + 9 + 15 + \cdots + 123 + 129 + 135[/tex]

Reverse the order of terms:

[tex]S = 135 + 129 + 123 + \cdots + 15 + 9 + 3[/tex]

If we add up the terms in the same positions, we get twice [tex]S[/tex] on the left side, while on the right side we observe that each pair of terms will sum to 138.

[tex]S + S = (3 + 135) + (9 + 129) + (15 + 123) + \cdots + (135 + 3)[/tex]

[tex]2S = 138 + 138 + 138 + \cdots + 138[/tex]

and since there are 24 terms in the sum, the right side is the sum of 24 copies of 138. In other words,

[tex]2S = 24 \times 138[/tex]

and solving for [tex]S[/tex] gives

[tex]S = \dfrac{24\times138}2 = \boxed{1656}[/tex]


The sin (0) = - and lies in Quadrant III. Find the exact values of the sine
6'
and cosine of 20.

Answers

Answer:

Can you give exanation

I don't understand

4. Write the function of a linear equation that includes the
points (1,3) and (2,9).

Answers

Answer:

y = 6x - 3

Step-by-step explanation:

the equation of a line in slope- intercept form ( linear function ) is

y = mx + c ( m is the slope and c the y- intercept )

calculate m using the slope formula

m = [tex]\frac{y_{2}-y_{1} }{x_{2}-x_{1} }[/tex]

with (x₁, y₁ ) = (1, 3 ) and (x₂, y₂ ) = (2, 9 )

m = [tex]\frac{9-3}{2-1}[/tex] = [tex]\frac{6}{1}[/tex] = 6 , then

y = 6x + c ← is the partial equation

to find c substitute either of the 2 points into the partial equation

using (1, 3 ) , then

3 = 6 + c ⇒ c = 3 - 6 = - 3

y = 6x - 3 ← equation of linear function

PLEASE ANSWER ASAP!!!!!! WILL GIVE BRAINLIEST!!!

Answers

Answer ya so  1 is the answer

Step-by-step explanation:

Answer:

1

Step-by-step explanation:

Given expression:

[tex]\sf \left(\dfrac{3a^{-2}b^6}{2a^{-1}b^5} \right)^2[/tex]

To find the value of the expression when a = 3 and b = -2, substitute these values into the expression:

[tex]\implies \sf \left(\dfrac{3(3)^{-2}(-2)^6}{2(3)^{-1}(-2)^5} \right)^2[/tex]

[tex]\textsf{Apply exponent rule} \quad a^{-n}=\dfrac{1}{a^n}[/tex]

[tex]\sf \implies \left(\dfrac{3\left(\dfrac{1}{3^2}\right)(-2)^6}{2\left(\dfrac{1}{3^1} \right)(-2)^5} \right)^2[/tex]

[tex]\sf \implies \left(\dfrac{3\left(\dfrac{1}{9}\right)(-2)^6}{2\left(\dfrac{1}{3} \right)(-2)^5} \right)^2[/tex]

[tex]\sf \implies \left(\dfrac{\left(\dfrac{3}{9}\right)(-2)^6}{\left(\dfrac{2}{3} \right)(-2)^5} \right)^2[/tex]

[tex]\sf \implies \left(\dfrac{\left(\dfrac{1}{3}\right)(-2)^6}{\left(\dfrac{2}{3} \right)(-2)^5} \right)^2[/tex]

[tex]\textsf{Apply exponent rule} \quad (-a)^n=a^n,\:\: \textsf{ if }n \textsf{ is even}[/tex]

[tex]\textsf{Apply exponent rule} \quad (-a)^n=-a^n,\:\: \textsf{ if }n \textsf{ is odd}[/tex]

[tex]\sf \implies \left(\dfrac{\left(\dfrac{1}{3}\right) (2^6)}{\left(\dfrac{2}{3} \right) (-(2^5))} \right)^2[/tex]

[tex]\sf \implies \left(\dfrac{\left(\dfrac{1}{3}\right) (64)}{\left(\dfrac{2}{3} \right) (-32)} \right)^2[/tex]

[tex]\sf \implies \left(\dfrac{\left(\dfrac{1 \times 64}{3}\right)}{\left(\dfrac{2 \times -32}{3} \right)} \right)^2[/tex]

[tex]\sf \implies \left(\dfrac{\dfrac{64}{3}}{\dfrac{-64}{3}} \right)^2[/tex]

When dividing fractions, flip the second fraction and multiply it by the first:

[tex]\implies \sf \left( \dfrac{64}{3} \times \dfrac {3}{-64} \right)^2[/tex]

[tex]\implies \sf \left( \dfrac{64 \times 3}{3 \times (-64)}\right)^2[/tex]

[tex]\implies \sf \left( \dfrac{192}{-192}\right)^2[/tex]

[tex]\implies \sf \left(-1\right)^2[/tex]

[tex]\textsf{Apply exponent rule} \quad (-a)^n=a^n,\:\: \textsf{ if }n \textsf{ is even}[/tex]

[tex]\sf \implies 1^2=1[/tex]

The vertex of this parabola is at (3-2). When the value is 4 the
y-value is 3. What is the coefficient of the squared expression in the
parabola's equation?
A. 7
B. 1
C. -1
D. 5

Answers

Answer:

D

Step-by-step explanation:

the equation of a parabola in vertex form is

y = a(x - h)² + k

where (h, k ) are the coordinates of the vertex and a is a multiplier

here (h, k ) = (3, - 2 ) , then

y = a(x - 3)² - 2

to find a substitute the point (4, 3 ) into the equation

3 = a(4 - 3)² - 2 ( add 2 to both sides )

5 = a × 1² = a

the coefficient of the squared expression is 5

(8.6x107)-(9.1x10-8)simplify

Answers

Answer:

  85,999,999.999 999 909

Step-by-step explanation:

The expression represents the difference of a relatively large number and one that is relatively small. That difference is approximately the value of the large number. The exact value requires 17 digits for its proper expression. Most calculators and spreadsheets cannot display this many digits.

Standard form

The numbers in standard form are ...

  86,000,000 = 8.6×10^7

  0.000000091 = 9.1×10^-8

Difference

Their difference is ...

  86,000,000 -0.000000091 = 85,999,999.999 999 909

In scientific notation, this is ...

  8.599 999 999 999 990 9×10^7

f(x)=x^2. which of these is g(x) ?

a. g(x)=(1/5x)^2
b. g(x)=5x^2
c.g(x)=(1/4x)^2
d.g(x)=1/5x^2

Answers

Answer:

a

Step-by-step explanation:

we know that g(x) means y therefore in this case let's suppose we have the coordinate (5;y) so we must find the corresponding y value

*DO THE TRIAL AND ERROR METHOD*

take any equation above and substitute the value of x of which is 5 and the corresponding value should be 1.

[tex]g(x) = (\frac{1}{5} x) {}^{2} [/tex]

A basketball coach recorded how many shots each of the 30 players at tryouts made in a set of 6 attempts.

Answers

Answer:

Step-by-step explanation:

what is the question?

Answer:

Median = A

Mean = B

Step-by-step explanation:

Khan Academy

Carolyn bought two brands of socks. She bought a total of 5 pairs. One brand cost $2 each and the other brand cost $3 each. She spent $13. Find the number of pairs each brand of socks she bought

Answers

Answer:

Step-by-step explanation:

Declaration

Let Brand A number of 2.00 socks= x

Let Brand B number of 3.00 socks = y

Equations

Number of socks : x + y = 5

Cost of socks: 2x + 3y = 13

Solution

Multiply top equation by 2

2(x+ y = 5)

2x + 2y = 10

Write the new equation under the cost of socks equation

2x + 3y = 13

2x + 2y = 10       Subtract.

y = 3

She spent 3 dollars on 1 pair of socks.

Now she needs to calculate the number of 2.00 dollar socks

2x + 3y = 13

2x + 3*3 = 13                Combine the left

2x + 9 = 13                   Subtract 9 from both sides.

2x +9-9 = 13-9             Combine

2x = 4                          Divide by 2

2x/2 = 4/2

x = 2

Answer

#2 dollar socks = 2

#3 dollar socks = 3

An architect is designing new housing structures for the primate section at the zoo. Her plan is shown below. Which animals will live in a building that is similar to the main primate house? A orangutans B chimpanzees C gibbons D gorillas

Answers

Answer:

A

Step-by-step explanation:

:p

A sequence is defined by the recursive function f(n + 1) = one-half(n). If f(3) = 9 , what is f(1) ?

Answers

The value of f(1) is 81

What is Recursive function?

Recursive Function is a function that repeats or uses its own previous term to calculate subsequent terms and thus forms a sequence of terms

Given:

f(n + 1) = 1/3* f(n)

f(3) = 9

let n=0

f(0+1) = 1/3 * f(0)

f(1) = 1/3 * f(0)

Let n=1

f(1+1) = 1/3 * 1/3*f(0)

f(2) = 1/9 * f(0)

let n=2

f(2+1) = 1/3 * 1/9* f(0)

f(3) = 1/27 * f(0)

Also, f(3) =9

So,

9 = 1/27 * f(0)

f(0) = 27 * 9

f(0) = 243

Now,

f(1) = 1/3 * f(0)

f(1) = 1/3/ 243

f(1) = 81

Learn more about this concept here:

https://brainly.com/question/14216181

#SPJ1

What function is shown in the graph above?

Answers

By definition of linear functions and the comparison with the attached figure, the function that represents the graph is y = (7/15) · x + 4, - 6 ≤ x ≤ 9.

What kind of function represents the graph?

Graphically speaking, linear functions represent lines and we see that the line seen in the figure presents two bounds, the points (- 6, 0) and (9, 7). Linear functions are characterized by slope and intercept:

y = m · x + b     (1)

Slope

m = (7 - 0)/[9 - (- 6)]

m = 7/15

Intercept

b = 4

By definition of linear functions and the comparison with the attached figure, the function that represents the graph is y = (7/15) · x + 4, - 6 ≤ x ≤ 9.

To learn more on linear functions: https://brainly.com/question/14695009

#SPJ1

Find the HCF of
x^4 y^2 and x^3 y^3

Answers

Here's your answer down here↓:

Step-by-step explanation:

x^3 - y^3 = (x - y)(x^2 + xy + y^2)

(x^4 - y^4) = (x^2 - y^2)(x^2 + y^2) = (x - y)(x + y)(x^2 + y^2)

Ok. So the factor (x-y) appears once in the top line and once in the second line. So we are going to take it the least amount of times.

So, the factor (x + y) appears in the top line zero times and in the second line one time so we will take it where it appears the least which is zero times so we are still at (x - y)

And it Same goes for the factors of (x^2 + y^2) and (x^2 + xy + y^2)

if x + 3y = 25 write y in terms of x and also find the two solutions of this equation

Answers

Answer:

y= 25/3 - x/3

Step-by-step explanation:

y= 25/3 - x/3

Give ur answer in standard form.
3 x 10^4 ÷6 x 10^-4

Answers

Answer:

5.0×10⁷

Step-by-step explanation:

(3÷6)×(10⁴÷10^-4)

(0.5)×(10⁸)

5.0×10^-1×10⁸

5.0×10⁷

[tex] \frac{3 \times {10}^{4} }{6 \times {10}^{ - 4} } \\ \\ \frac{1 \times {10}^{4 - ( - 4)} }{2 } \\ \\ \frac{ 1 \times {10}^{4 + 4} }{2} \\ \\ \frac{1 \times {10}^{8} }{2} \\ \\ 0.5 \times {10}^{8} .[/tex]

What is the equation of the line that passes through the point (5,-2) and has a
slope of -2/5

Answers

-------------------------------------------------------------------------------------------------------------

Answer:  [tex]\textsf{y = -2/5x}[/tex]

-------------------------------------------------------------------------------------------------------------

Given:  [tex]\textsf{Passes through (5, -2) and slope of -2/5}[/tex]

Find:  [tex]\textsf{The equation that follows the details provided}[/tex]

Solution: We first need to plug into the point-slope form and after simplifying, distributing, and solving for y we will complete our equation.

Plug in the values

[tex]\textsf{y - y}_1\textsf{ = m(x - x}_1\textsf{)}[/tex][tex]\textsf{y - (-2) = -2/5(x - 5)}[/tex]

Distribute and simplify

[tex]\textsf{y + 2 = -2/5(x - 5)}[/tex][tex]\textsf{y + 2 = (-2/5 * x) + (-2/5 * (-5))}[/tex][tex]\textsf{y + 2 = -2/5x + 2}[/tex]

Subtract 2 from both sides

[tex]\textsf{y + 2 - 2 = -2/5x + 2 - 2}[/tex][tex]\textsf{y = -2/5x + 2 - 2}[/tex][tex]\textsf{y = -2/5x}[/tex]

Therefore, the final equation that follows the information that was provided is y = -2/5x.

Other Questions
state one example of 2 numbers where the LCM of the 2 numbers is the product of those numbers. explain your answers What is Anna in this sentence?The class made Anna president. You are the network administrator for a small organization. recently, you contracted with an isp to connect your organization's network to the internet to provide users with internet access. since doing so, it has come to your attention that an intruder has invaded your network from the internet on three separate occasions. what type of network hardware should you implement to prevent this from happening again During the Reagan Administration of the 1989s , what controlling image was used to harm Black women ? The root mal means bad or evil. Can you defermine what the word maleficent means by usingthe root? How many atoms from the following molecule will be detected in a 13c nmr experiment? we are not asking for unique signals, just total number of atoms What statement do you begin with when writing the first draft? a. a main idea statement b. a summary statement c. a closing statement d. an ending statement State and explain four ways One can organise his or her artworks write a quadratic function in standard form with zeros -5 and 0 Investing in servers for extended periods of time, such as committing to use servers for three to five years, works well when? * 05.02 MC)Angie is working on solving the exponential equation 34^x= 23; however, she is not quite sure where to start. Using complete sentences, describe to Angie how to solve this equation. Which word is the best antonym for stalwart. a. unendingb. immobilec. weak 1. Draw an appropriate Venn diagram to depict each of the following sets. U = The set of integers. A = The set of even integers. B = The set of odd integers. C = The set of multiples of 3. D= The set of prime numbers. 1. Draw an appropriate Venn diagram to depict each of the following sets . U = The set of integers . A = The set of even integers . B = The set of odd integers . C = The set of multiples of 3 . D= The set of prime numbers . when the price of a product falls, demand for its substitute will _______. multiple choice question. stay the same decrease increase Describe a scenario where the function value exists at x=c, but the limit does not exist. Belgium returned a gold _____ belonging to late prime minister patrice lumumba of the democratic republic of the congo. Use the accompanying table to answer the questions regarding money ratios.(a) What is the capital a 40-year-old should have with an income of $32,000?(b) How much should a 35-year-old be saving for retirement with an income of $86,000?(c) How much in education debt should a 30-year-old have with an income of $59,000?The 40-year-old should have $___in capital.(Simplify your answer)(b) The 35-year-old should be saving $ for retirement.(Simplify your answer)(c) The 30-year-old should have $___ in education debt.(Simplify your answer). You are on a boat in a fog and know there are cliffs ahead of you somewhere, but you cannot see them. you use your fog horn to send out a blast of sound (assume the temperature of the air is 20 oc) and the time it takes for the sound to return to you is 20.0 s. how far away are the cliffs (in metric units) A sudden and widespread disruption of financial markets that occurs when people lose faith in the liquidity of financial institutions and markets is a(n): a nurse is caring for a newborn with fetal alcohol spectrum disorder. what characteristic of the fetal alcohol spectrum disorder should the nurse assess for in the newborn?